There are 5 slices of pepperoni pizza, 1 slice of sausage pizzá, and 3 slices of cheese pizza left at the pizza party. Without looking, Amy took a slice of pizza, ate it, and then took another slice. What is the probability of Amy eating two slices of cheese pizza?

Answers

Answer 1

Answer:

3/8

Step-by-step explanation:

add 5+1+3=9

and there is 3 cheese pizza so its 3 over 8.

Answer 2

Answer:

3/9

Step-by-step explanation:


Related Questions

if a line is perpendicular to y=-2/3x+2, will it be perpendicular to 2x+3y=12?

Answers

Answer:

3x + 2y = -2

Step-by-step explanation:

Given Equation is − 2 x  +  3 y  =  12

3 y  = 2 x  +  12

y =  ( 2 /3 )  y  +  12

Slope of this line is  m  =  2/ 3

Slope of Line A m a  =  2

Slope of Line B  m b  =  2/ 3

Slope of Line C  m c  =  − ( 2 /3 )

Slope of Line D  m d  =  −  ( 3 /2 )

since  m d  =  − 1 /m  ,  D  is perpendicular to the given line

What if f(x)=(1/7)(7x), what is f(3)

Answers

Answer:

  3

Step-by-step explanation:

f(x) = (1/7)(7x) can be simplified to ...

f(x) = x

Then for x=3, ...

  f(3) = 3

Which one of the following is not a fixed cost in owning a vehicle?
1 point
Gas
Insurance
Registration
Tags

Answers

Step-by-step explanation:

tags is not a fixed cost

Needing help can’t seem to find the answers? ‍♀️

Answers

Step-by-step explanation:

19. Kim's Path = √240² + 100²

= √57600 + 10000

= √67600

= 260 ft.

Answer:

17 question: we are going to use Pythagorean Theorem :

33^2 + 56^2 = x^2

1089+3165 = x^2

4225=x^2

x=-65

x=65

the answer is c)65

18 question i dont know :((

19 question : we are going to use same method as the first question : the answer is 260 ft

that's all that i can do for you

Step-by-step explanation:

hope this helps you :)

29. Find the circumference.
11
a. 3.14
b. 69.08
c. 121

Answers

Answer:

b.69.08

Step-by-step explanation:

formula:

c=2(pi)r

c=2(pi)11

Answer:b

Step-by-step explanation:

radius=r=11

π=3.14

Circumference=2 x π x r

Circumference=2 x 3.14 x 11

Circumference=69.08

Nora made 15 gallons of lemonade for a community picnic.

Part A
Which of the following can be used to find the number of pints of lemonade that Nora made?

15 × 2 × 2
15 × 4 × 2
15 × 3 × 2
15 × 4 × 3

Part B
How many pints of lemonade did Nora make?
Enter your answer in the box.

pints

Answers

Answer:

I also belive for Part A the second one is the answer because it equals 120 Part B is 120 pints! Good Luck! And I hope this Helps!

Answer:

B

Step-by-step explanation:

BRAINLIEST ASAP! LENGTH OF AC?

Answers

Answer:

2.33 units

Step-by-step explanation:

[tex]\tan 25\degree =\frac{AC}{5}\\\\0.46630 = \frac{AC}{5}\\\\AC = 0.46630 \times 5\\AC =2.3315\\AC = 2.33 \: units[/tex]

Which equation represents the hyperbola shown in the
graph?
10
8
(x - 2)2
(y + 3)
25
(-2,5) 6
(-7,3)
(1-21)
4-12-10 -8 -6 4-2
(3,3)
(x + 2)2
(y = 3) = 1
4
2 4 6
(x + 2)2
25
(y - 3)2
4
1
(232) - (7,31 = 1
(x - 2)2
25

Answers

Answer:

Step-by-step explanation:

A general equation of a hyperbola is

 x^2     y^2

-------- - ------- = 1   (This applies only when the center of the hyperbola

 a^2     b^2                               is at (0, 0)  ).

You must compare the given equations to this standard form to identify which represents the hyperbola shown, and also you must share the illustration of the hyperbola.

The equation represents the hyperbola shown in the graph is  (y - 1)² / 9 - (x + 4)² / 4  = 1

What is a hyperbola?

A hyperbola is a type of smooth curve lying in a plane, defined by its geometric properties or by equations for which it is the solution set.

A hyperbola has two pieces, called connected components or branches, that are mirror images of each other and resemble two infinite bows.

Given that, a graph, showing hyperbola, we need to find the equation,

We have the general equation for up - down facing hyperbola as

(y - k)² / b²- (x - h)² / a²  = 1.

Let's start listing the properties of this graph -

Taking a look at the graph we see that the center point of our hyperbola here is (- 4, 1).

Therefore, (h, k) = (- 4,1).

This is the semi distance from the center to one of the vertices. Here it will be the distance from points (- 4,1) and (- 4,4) or 3 unit difference.

Therefore, a = 3.

That gives asymptotes. Now remember that it will be in the form

y = ± b / a.

We already know a = 3, so we have to find b.

Looking at this graph we can say that another point besides (- 4,1) that lies on the "dotted line" is (- 2, - 2).

Calculating the slope of the dotted line would be as follows,

Given: (- 4,1) and (- 2, - 2)

Slope = - 2 + 4 / - 2 - 1 = 2 / 3

We have the equation y = 2 / 3x.

Therefore, b = 2.

Let's substitute to equation...

h = - 4, k = 1, b = 2, a = 3

(y - 1)² / (3)²- (x + 4)² / (2)²  = 1

(y - 1)² / 9 - (x + 4)² / 4  = 1

Hence, the equation represents the hyperbola shown in the graph is  (y - 1)² / 9 - (x + 4)² / 4  = 1

Learn more about hyperbolas, click;

https://brainly.com/question/28989785

#SPJ7

The complete question is attached

Please help ASAP! I will mark BRAINLIEST! Please answer CORRECTLY! No guessing! CHECK ALL THAT APPLY

Answers

Answer:

D. 96

F. 32

Step-by-step explanation:

The inequality 3x ≥ 96, means that x and 3 must be multiplied to get a product that is greater than 96.  So to figure this out, you must plug in the numbers given for x, like so:

3x ≥ 96

A. 3(31) ≥ 96

     93 ≥ 96                            This is incorrect since 93 is not greater than 96

A tip that makes this easier is that, you can see that 31 multiplied by 3 is 93, which is only a couple numbers off 96, so all numbers after 31 will result in a product greater than 96. For example:

D. 3(96) ≥ 96

    288 ≥ 96                             This inequality is correct

F. 3(32) ≥ 96

   96 ≥ 96                        

The inequality sign states that the number must be either greater or equal to 96, which it is.

Find the mean, median, mode and range for each set of data. Calculator usage is encouraged!
1. 23, 87, 19, 34, 37, 87, 81, 5, 14, 100, 26 Please help thank you!

Answers

Answer:

mean: 46.63636

median: 34

mode: 87

range:95

How To:

Step 1 : To find Mean

    Average = ( 1 + 5 + 5 + 7 + 8 + 10 ) / 6

             =36 / 6

    Mean = 6

Step 2 : To find Median

   Middle value = ( 5 + 7 ) / 2

               = 12 / 2

    Median = 6

Step 3 : To find Mode

   Mode = 5 (The number with more repetition, here 5 is repeated two times)

Step 4 : To find Range

    Range = Largest number - Smallest number

       = 10-1

       = 9

Range = 9

Answer:

Mean: 46.6

Mode: 87

Median: 34

Range: 95

Step-by-step explanation:

Mean: (finding the average)

Median: (the middle number of the data set)

Mode: (the most number repeated from the data set)

Range: (is the difference between the highest value and the lowest value)

first arrange the following data set.

23, 87, 19, 34, 37, 87, 81, 5, 14, 100, 26

so:

5, 14, 19, 23, 26, 34, 37, 81, 87, 87, 100

Lets us first find the mean by adding up all the numbers and dividing it by the amount of numbers in the data set.

Mean: 5 + 14 + 19 + 23 + 26 + 34 + 37 + 81 + 87 + 87 + 100 = 513/11 = 46.6

Mode: 87

Median: 34

Range: 100 - 5 = 95

Ellis is 6 feet tall. If she currently has a 4 foot shadow, what angle are the sunrays making with the ground? Round decimal to the nearest hundredth.

Answers

Answer:

19.70

Step-by-step explanation:

For this case we have the following trigonometric relationship:

Tan (x) = (4/6)

Clearing the angle we have:

x = Atan (4/6)

x = 19.77

Rounding off we have:

x = 19.70 degrees

A store sells gift cards in preset amounts. You can purchase gift cards for ​$20 or ​$30. You have spent ​$680 on gift cards. Write an equation in standard form to represent this situation. What are three combinations of gift cards you could have​ purchased?
Let x be the number of gift cards for ​$20​, and let y be the number of gift cards for ​$30. Write an equation in standard form to represent this situation

Answers

Answer:

20x + 30y = 680

Step-by-step explanation:

Give me a good rating please!

An equation is a mathematical statement that is made up of two expressions connected by an equal sign.

The equation represents the number of $20 and $30 gift cards bought.

20x + 30y = 680

What is an equation?

An equation is a mathematical statement that is made up of two expressions connected by an equal sign.

We have,

Number of gift cards for $20 = x

Number of gift cards for $30 = y

Total amount spend on gift card = $680

The equation represents the number of $20 and $30 gift cards bought.

20x + 30y = 680

Thus,

The equation represents the number of $20 and $30 gift cards bought.

20x + 30y = 680

Learn more about equations here:

https://brainly.com/question/17194269

#SPJ2

A 5000​-seat theater has tickets for sale at ​$28 and ​$40. How many tickets should be sold at each price for a sellout performance to generate a total revenue of ​$153 comma 200​?

Answers

Answer:

Let's denote:

x: number of ticket 28$

y: number of ticket 40$

Then, we have:

x + y =5000

28x + 40y = 153200

=> 28(5000 - y) + 40y = 153200

=> 12y = 153200 - 140000

=> 12y =13200

=> y = 1100 (ticket 40$)

=> x = 5000 - 1100 = 3900 (ticket 28$)

A bag contains 3 red marbles and 6 blue marbles. A second bag contains 6 green marbles and 4 yellow marbles. You choose a marble from bag A and then a marble from bag B, what would be the probability of selecting one blue marble and one yellow marble?

Answers

Answer:

4/15

Step-by-step explanation:

Bag A

3 red marbles and 6 blue marbles.  = 9 marbles

P(blue) = blue/total =6/9 = 2/3

Bag B

6 green marbles and 4 yellow marbles.  = 10 marbles

P(yellow) = yellow/total=4/10 = 2/5

P(blue,yellow) = 2/3 * 2/5 = 4/15

What is the slope of the line

Answers

Answer:

4/5

Step-by-step explanation:

There are so many points on the graph whose coordinates are impossible to estimate. Look for two "nice" points--points with integer coordinates. The line passes through (-2, 0) and (3, 4).

Slope = rise / run. It's the ratio of vertical change to horizontal change as you move from one point on the line to another.

Moving from (-2, 0) to (3, 4), the vertical change is 4 - 0 = 4.

The horizontal change is 3 - (-2) = 5.

The slope is 4/5

What’s the correct answer for this?

Answers

I’m pretty sure it’s B.

According to a Pew Research Center report from 2012, the average commute time to work in California is 27.5 minutes. To investigate whether the small city she lives in has a different average, a California high school student surveys 45 people she knows (her teachers, her parents, and their friends and co-workers) and finds the average commute time for this sample to be 24.33 minutes with a standard deviation of 9.53 minutes. The data are not too skewed. The null and alternative hypotheses of her study are: H0 : µ = 27.5 versus Ha : µ 6= 27.5

Required:
a. Identify the observational units for this study.
b. Identify the variable of interest and state whether it is categorical or quantitative.
c. Identify (in words and using an appropriate symbol) the parameter of interest
d. Use the 2SD approach to find a 95% confidence interval for the parameter.
e. Interpret the interval from part d. in context.

Answers

The answer is maybe given if you please summarize the question some. Thanks!

In the right triangle shown DF=EF=3. How long is DE?

Answers

Answer:

4.24

Step-by-step explanation:

To solve this, use the Pythagorean therom. A^2 + b^2 = C^2

in this case a = 3  and b = 3

so 9 + 9 =  sqrt 18  

4.24

Answer:3√(2)

Step-by-step explanation:

DF=3

EF=3

DE=√(3^2 + 3^2)

DE=√(3x3 + 3x3)

DE=√(9+9)

DE=√(18)

DE=√(2 x 9)

DE=√(2) x √(9)

DE=√(2) x 3

DE=3√(2)

Need help on this assignment it is due VERY SOON

All have something to do with area of composite figures. Please help. I'd be very greatful.

Answers

Answer

the first photo answer is 174 (2nd photo is A) Answer to 3rd photo is 72. The last photo is 8100 ft²

Step-by-step explanation:

1st Photo

12 x 12 is 144

5 times 12 divided by 6 - (5 x 6) 30

144 plus 30 is 174

2nd Photo

3 x 5 15

15 is A

3rd Photo

6 x 12 is 72

72 for 3rd Photo

4th Photo

90 x 90 is

8,100

What is the sixth term in the sequence 3, 6, 11, 18

Answers

Answer:

38

Step-by-step explanation:

3 to 6 is an add by 3 then 6 to 11 is an add by 5 then 11 to 18 is an add by 7 each add is increased by 2 so the next add will be 9 giving you 27 then you will add 11 because it's increased by two giving you 38

For the data 20, 40, 50, 20, 10, 70. What is there mean absolute deviation?

Answers

Answer:

18.333

Step-by-step explanation:

To generate leads for new business, Gustin Investment Services offers free financial planning seminars at major hotels in Southwest Florida. Gustin conducts seminars for groups of 25 individuals. Each seminar costs Gustin $3,500, and the commission for each new account opened is $5,000. Gustin estimates that for each individual attending the seminar, there is a 0.01 probability that he/she will open a new account.

(a) Determine the equation for computing Gustin's profit per seminar, given values of the relevant parameters. Profit = (New Accounts Opened × ) –

(b) What type of random variable is the number of new accounts opened? (Hint: Review Appendix 11.1 for descriptions of various types of probability distributions.)

(c) Choose the appropriate spreadsheet simulation model to analyze the profitability of Gustin's seminars. (I) (II) (III) (IV) Would you recommend that Gustin continue running the seminars?

(d) How many attendees (in a multiple of five, i.e., 25, 30, 35, . . .) does Gustin need before a seminar's average expected profit is greater than zero?

Answers

Answer:

a) profit = (new account opened x 5000) -3500

b) Opening account is binomial distribution with n =25 and p = 0.01

c) Probability of loss is 0.77781 --I don't recommend the company that it running the seminar

d) n ≅ 71

Step-by-step explanation:

See attached image

Simplify the expression below.
14a8y3 - 7 Ay5 + 28a12y2
7aty
A.
OB.
2a²y3 - ay5 + 4a3y2
2a4y? - JA + 428 y
2a4y3 – 5 + 428 y?
D. 2012,4 - 2876 +4215,3
C.

Answers

Answer:

14a8y3 - 7 Ay5 + 28a12y2-  7ay2 • (4a11 + 2a7y - y3)

Step-by-step explanation:

Equation at the end of step  1  :

 (((14•(a8))•(y3))-(7a•(y5)))+((22•7a12)•y2)

Step  2  :

Equation at the end of step  2  :

 (((14•(a8))•(y3))-7ay5)+(22•7a12y2)

Step  3  :

Equation at the end of step  3  :

 (((2•7a8) • y3) -  7ay5) +  (22•7a12y2)

Pull out like factors

Answer:   7ay2 • (4a11 + 2a7y - y3)

Hope this helps.

A teacher of statistics wants to know if a new teaching methodology that includes IT is efficient in terms of increased average score. He took a class with old methodology and a class with new methodology for samples and gave a same test. Open the file by clicking the file name above. Once you open the file and run Excel, you need not open it again. What is Ha? Find it from Excel output that you generate.
a) 0.62.
b) 0.5.
c) 0.31.
d) -0.5.

Answers

Answer:

The answer is 0.31

Step-by-step explanation:

Old Method                                       New Method                          .

Mean                      73.5625                  Mean            75.70588

Standard Error        3.143736             Standard Error  2.923994

Median                    72                          Median            75

Mode                       90                          Mode               64

Standard deviation  12.57494           Standard deviation 12.05594

Sample Variance     158.1292            Sample Variance     145.3456

Kurtosis                     -1.14544              Kurtosis                   -0.76646

Skewness                 0.171025             Skewness                0.091008

Range                       39                        Range                      41

Minimum                   55                        Minimum                 56

Maximum                  94                        Maximum                 97

Sum                          1177                        Sum                        1287

Count                         16                         Count                      17

State the hypotheses. The first step is to state the null hypothesis and an alternative hypothesis.

Null hypothesis: μNew< μOld

Alternative hypothesis: μNew > μOld

Note that these hypotheses constitute a one-tailed test. The null hypothesis will be rejected if the mean difference between sample means is too small.

Formulate an analysis plan. For this analysis, the significance level is 0.05. Using sample data, we will conduct a two-sample t-test of the null hypothesis.

Analyze sample data. Using sample data, we compute the standard error (SE), degrees of freedom (DF), and the t statistic test statistic (t).

[tex]SE=\sqrt{\frac{S_1^2}{n_1} +\frac{S_2^2}{n_2} } \\\\SE=4.29[/tex]

DF = 31

[tex]t = \frac{(x_1-x_2)-d}{SE} \\\\t = - 0.4997[/tex]

where s1 is the standard deviation of sample 1, s2 is the standard deviation of sample 2, n1 is the size of sample 1, n2 is the size of sample 2, x1 is the mean of sample 1, x2 is the mean of sample 2, d is the hypothesized difference between population means, and SE is the standard error.

The observed difference in sample means produced a t statistic of - 0.499. We use the t Distribution Calculator to find P(t < - 0.499) = 0.311

Therefore, the P-value in this analysis is 0.311.

Interpret results. Since the P-value (0.311) is greater than the significance level (0.05), we cannot reject the null hypothesis.

From the above test we do have sufficient evidence in the favor of the claim that new method is efficient than the old method.

Oil is leaking from an oil tanker, and an expanding circle of oil is spreading on the ocean. The radius, r, of

modeled by the function r(s)=315, where sis time in seconds.

The area of the spill when s=5 seconds is

1 square inches.

Reset

Reset

Next

Next

Answers

Answer:

[tex]45\pi$ square inches[/tex]

Step-by-step explanation:

The radius, r of the expanding circle of oil is modeled by the function:

[tex]r=3\sqrt{s}[/tex] , where s is time in seconds.

When s=5

Radius [tex]r(5)=3\sqrt{5}$ inches[/tex]

Area of a circle [tex]=\pi r^2[/tex]

Therefore, the area of the oil spill when s=5 seconds

[tex]=\pi* (3\sqrt{5})^2\\=45\pi$ square inches[/tex]

The area of the spill when s=5 seconds is 45pi square inches.

A university found that 20% of its students withdraw without completing the introductory statistics course. Assume that 20 students registered for the course. (Round your answers to four decimal places.) (a) Compute the probability that 2 or fewer will withdraw. (b) Compute the probability that exactly 4 will withdraw. (c) Compute the probability that more than 3 will withdraw. (d) Compute the expected number of withdrawals.

Answers

Answer:

(a) Probability that 2 or fewer will withdraw is 0.2061.

(b) Probability that exactly 4 will withdraw is 0.2182.

(c) Probability that more than 3 will withdraw is 0.5886.

(d) The expected number of withdrawals is 4.

Step-by-step explanation:

We are given that a university found that 20% of its students withdraw without completing the introductory statistics course.

Assume that 20 students registered for the course.

The above situation can be represented through binomial distribution;

[tex]P(X =r) = \binom{n}{r} \times p^{r} \times (1-p)^{n-r};x=0,1,2,3,......[/tex]

where, n = number of trials (samples) taken = 20 students

            r = number of success  

            p = probability of success which in our question is probability  

                  that students withdraw without completing the introductory  

                  statistics course, i.e; p = 20%

Let X = Number of students withdraw without completing the introductory statistics course

So, X ~ Binom(n = 20 , p = 0.20)

(a) Probability that 2 or fewer will withdraw is given by = P(X [tex]\leq[/tex] 2)

P(X [tex]\leq[/tex] 2) =  P(X = 0) + P(X = 1) + P(X = 2)

=  [tex]\binom{20}{0} \times 0.20^{0} \times (1-0.20)^{20-0}+ \binom{20}{1} \times 0.20^{1} \times (1-0.20)^{20-1}+ \binom{20}{2} \times 0.20^{2} \times (1-0.20)^{20-2}[/tex]

=  [tex]1 \times1 \times 0.80^{20}+ 20 \times 0.20^{1} \times 0.80^{19}+ 190\times 0.20^{2} \times 0.80^{18}[/tex]

=  0.2061

(b) Probability that exactly 4 will withdraw is given by = P(X = 4)

                      P(X = 4) =  [tex]\binom{20}{4} \times 0.20^{4} \times (1-0.20)^{20-4}[/tex]

                                 =  [tex]4845\times 0.20^{4} \times 0.80^{16}[/tex]

                                 =  0.2182

(c) Probability that more than 3 will withdraw is given by = P(X > 3)

P(X > 3) =  1 - P(X [tex]\leq[/tex] 3) = 1 - P(X = 0) - P(X = 1) - P(X = 2) - P(X = 3)

=  [tex]1-(\binom{20}{0} \times 0.20^{0} \times (1-0.20)^{20-0}+ \binom{20}{1} \times 0.20^{1} \times (1-0.20)^{20-1}+ \binom{20}{2} \times 0.20^{2} \times (1-0.20)^{20-2}+\binom{20}{3} \times 0.20^{3} \times (1-0.20)^{20-3})[/tex]

=  [tex]1-(1 \times1 \times 0.80^{20}+ 20 \times 0.20^{1} \times 0.80^{19}+ 190\times 0.20^{2} \times 0.80^{18}+1140\times 0.20^{3} \times 0.80^{17})[/tex]

=  1 - 0.4114 = 0.5886

(d) The expected number of withdrawals is given by;

                        E(X)  =  [tex]n\times p[/tex]

                                 =  [tex]20 \times 0.20[/tex] = 4 withdrawals

BRAINLIST+11 Pts!!!
Jim just found a job with a take-home pay of $950 per month. He must pay $400 for rent and $100 for groceries each month. He also spends $100 per month on transportation. If he budgets $50 each month for clothing, $100 for restaurants and $50 for everything else, how long will it take him to save $1,800?

Answers

Answer:12 months

Step-by-step explanation:

400+400=800 so  950-800=150 1800/150=12

Bob, Paula and Sam invest $50000 in a business. Bob invests $4000 more than Paul does and Paul invests $5000 more than Sam does. If the total profit was $70000, select the correct answer. Note that the profit is distributed proportionally based on the respective amount each invested. A. The ratio of the investment of Bob, Paula and Sam is 11:15:10. B. The ratio of the investment of Bob, Paula and Sam is 12:17:21. C. The ratio of the investment of Bob, Paula and Sam is 12:5:4. D. The profit of Paula was $23,800

Answers

Answer:

D

Step-by-step explanation:

since sam invest the least, let a be the amount invested by sam

sam = a

paul = a + 5000

bob = a + 5000 + 4000

3a + 14000 = 50000

3a = 36000

a = 12000

thus sam is 12000, paul is 17000 and Bob is 21000

therefore the ratio of B:P:S is 21:17:12

profit by paula is 17/50 x 70000 = 23800

The profit by Paula is 17/50 x 70000 = 23800.

We have given that Bob, Paula and Sam invest $50000 in a business. Bob invests $4000 more than Paul does and Paul invests $5000 more than Sam does. If the total profit was $70000

Since sam invest the least, let a be the amount invested by sam

Therefore we get,

sam = a

What is the investment of Paul?

The investment of Paul = a + 5000

Bob = a + 5000 + 4000

3a + 14000 = 50000

3a = 36000

divide both sides by 3 so we get,

a=36000/3

a = 12000

Therefore, sam is 12000,

paul =5000+12000=17000 and

Bob =12000+9000= 21000

Therefore the ratio of B:P:S is 21:17:12

The profit by Paula is 17/50 x 70000 = 23800.

To learn more about the investement visit:

https://brainly.com/question/25300925


What can be determined by looking at the factored form of the polynomial f(x) = (x + 7)(x - 2) ?
A
The graph of the polynomial has zeros at (0,-7) and (0,2)
В
The graph of the polynomial has zeros at (7,0) and (-2,0)
C
The graph of the polynomial has zeros at (-7,0) and (2,0)
D
The graph of the polynomial has zeros at (0,7) and (0,-2)

Answers

Answer:

C . The graph of the polynomial has zeros at (-7,0) and (2,0)

Step-by-step explanation:

f(x) = (x + 7)(x - 2)

Zero's obtained:

(x+7)(x-2) = 0x+7 = 0 ⇒ x = -7x-2 = 0 ⇒ x = 2

Coordinates:

(-7, 0) and (2, 0)

Correct answer choice is:

C . The graph of the polynomial has zeros at (-7,0) and (2,0)

An angle whose measure is -102° is in standard position. Which quadrant does the terminal side of the angle fall?
Quadrant 1
Quadrant 2
Cuadrant 3
Cuadrant 4

Answers

Answer:3

Step-by-step explanation:

edg

Other Questions
kievs importance to Russian society can be best illustrated by Which of these conclusions is correct about P and Q in the diagram? P represents the land and Q represents the ocean. P represents the ocean and Q represents the land. P represents the land and Q represents the atmosphere. P represents the atmosphere and Q represents the ocean. Album covers are an effective way to:O A. indicate the high production values on the recording.O B. lend an air of respectability and trustworthiness to alternativemusicians.O c. get people who would otherwise download music to buy thealbumD. provide listeners with additional information about the music andmusicians.mm There are four countries in the region of Central Asia.Please select the best answer from the choices providedTF In which year did BEACH volleyball first become an olympic game [tex]3x + 2y + 2z = 19\\3x + y + z = 14\\[/tex]If the equations above are true, which of the following is the value of y + z?(A.) -5(B.) -4(C.) 0(D.) 4(E.) 5 What political causes has Buchanan focused on IS IT right to the left a b c d A typical cell phone uses wavelength of 0.36meters. How much energy does this wave have? damage to the skin and underlying tissues caused by prolonged exposure to extreme cold is called ________ Twenty-four 4-inch wide square posts are evenly spaced with 5 feet between adjacent posts to enclose a square field, as shown. What is the outer perimeter, in feet, of the fence? Express your answer as a mixed number. g On the first day of its fiscal year, Chin Company issued $10,000,000 of five-year, 7% bonds to finance its operations of producing and selling home improvement products. Interest is payable semiannually. The bonds were issued at a market (effective) interest rate of 8%, resulting in Chin receiving cash of $9,594,415. a. Journalize the entries to record the following: Issuance of the bonds. First semiannual interest payment. The bond discount is combined with the semiannual interest payment. (Round your answer to the nearest dollar.) Second semiannual interest payment. The bond discount is combined with the semiannual interest payment. (Round your answer to the nearest dollar.) If an amount box does not require an entry, leave it blank. 1. 2. 3. b. Determine the amount of the bond interest expense for the first year. $ c. Why was the company able to issue the bonds for only $9,594,415 rather than for the face amount of $10,000,000? The market rate of interest is the contract rate of interest. Therefore, inventors wi Hey guys I need help with this exercise can someone plz help me for 20 points?? Also write the explamation to your answer plzz!Math problem: 13 of the same cookbooks weigh 442 grams. What is the weight of 10 of these books? Use the grouping method to factor the polynomial below completely.3x3 + 9x2 + 4x + 12O A. (3.2 + 4)(x+4)O B. (3x2+3)(x + 4)O c. (3x2 + 4)(x+3)O D. (3x + 3)(x+3) There are 14 math teachers and 26 science teachers atschool today. What is the ratio of math teachers to mathand science teachers at school? discuss methods that individuals, groups and/ or the government have used since 1950 to deal with the inequality faced by african americans. help pls What does the questions how much? and how many? have in common? ATP molecule What molecule(s) is produced by these activities? The measure of an angle is 17. What is the measure of its complementary angle? 5(x-10)=30-15xOx=1Ox=4Ox=5Ox=8